2014 dxdy logo

Научный форум dxdy

Математика, Физика, Computer Science, Machine Learning, LaTeX, Механика и Техника, Химия,
Биология и Медицина, Экономика и Финансовая Математика, Гуманитарные науки




Начать новую тему Ответить на тему
 
 China TST 2004
Сообщение29.09.2008, 18:28 
Аватара пользователя


28/09/08
11
k \geq 2, 1 < n_1 < n_2 < \ldots < n_k и a,b-натуральные числа удовлетворяют условию
\prod^k_{i=1} \left( 1 - \frac{1}{n_i} \right) \leq \frac{a}{b} < \prod^{k-1}_{i=1} \left( 1 - \frac{1}{n_i} \right)
доказать неравенство \prod^k_{i=1} n_i \geq (4 \cdot a)^{2^k - 1}.

 Профиль  
                  
 
 
Сообщение29.09.2008, 20:58 


06/07/07
215
Интересно, а зачем здесь $b$?

 Профиль  
                  
 
 
Сообщение30.09.2008, 11:42 
Заслуженный участник


09/02/06
4382
Москва
Он наверное забыл сказать, что (a,b)=1 (a/b - несократимое представление рационального числа). Только всё равно утверждение в таком виде неверное. По видимому надо менять неравенство на противоположное, тогда о несократимости можно не волноваться.

 Профиль  
                  
 
 
Сообщение30.09.2008, 14:15 
Аватара пользователя


28/09/08
11
Вообще то наличие пробелов в условии очевидно. Потому я в точности скопировал условие, что бы посмотреть как его можно покрутить. Видимо неравенство действительно выполняеться в другую сторону( иначе не обьяснить отсутствия условий на a,b)

 Профиль  
                  
 
 
Сообщение03.10.2008, 05:40 
Аватара пользователя


28/09/08
11
Отличная новость: нашёл эту задачу в другом источнике,и предлагаю вам правильную формулировку
k \geq 2, 1 < n_1 < n_2 < \ldots < n_k и a,b-натуральные числа удовлетворяют условию
\prod^k_{i=1} \left( 1 - \frac{1}{n_i} \right) \leq \frac{a}{b} < \prod^{k-1}_{i=1} \left( 1 - \frac{1}{n_i} \right)
доказать неравенство \prod^k_{i=1} n_i \leq (4 \cdot a)^{2^k - 1}.

 Профиль  
                  
 
 
Сообщение03.10.2008, 08:33 


06/07/07
215
СОКАКУ писал(а):
Отличная новость: нашёл эту задачу в другом источнике,и предлагаю вам правильную формулировку
k \geq 2, 1 < n_1 < n_2 < \ldots < n_k и a,b-натуральные числа удовлетворяют условию
\prod^k_{i=1} \left( 1 - \frac{1}{n_i} \right) \leq \frac{a}{b} < \prod^{k-1}_{i=1} \left( 1 - \frac{1}{n_i} \right)
доказать неравенство \prod^k_{i=1} n_i \leq (4 \cdot a)^{2^k - 1}.
Для существования целых $a$, требуемых условием, необходимо и достаточно,
чтобы $\prod_{i=1}^{k-1}\left(1-\frac{1}{n_i}\right)-\prod_{i=1}^k\left(1-\frac{1}{n_i}\right)\geqslant\frac{1}{b}$,
или $\frac{1}{b}\leqslant\frac{1}{n_k}\prod_{i=1}^{k-1}\left(1-\frac{1}{n_i}\right)$, или $n_k\leqslant b\prod_{i=1}^{k-1}\left(1-\frac{1}{n_i}\right)$.
Очевидно, такие целые $a$ есть $0<b\prod_{i=1}^k\left(1-\frac{1}{n_i}\right)\leqslant a<b\prod_{i=1}^{k-1}\left(1-\frac{1}{n_i}\right)<b$, то есть они натуральные.

Тогда $\prod^k_{i=1}n_i \leqslant n_1\prod_{i=2}^{k}\left(b\prod_{j=1}^{i-1}\left(1-\frac{1}{n_j}\right)\right)\leqslant n_1\cdot b\left(1-\frac{1}{n_1}\right)a^{k-2}<$,
учитывая $n_1<n_2\leqslant b\left(1-\frac{1}{n_1}\right)$ продолжим
$<b^2\left(1-\frac{1}{n_1}\right)^2a^{k-2}\leqslant\frac{1}{\left(1-\frac{1}{n_2}\right)^2}a^k\leqslant$,
если $n_i$ - натуральные, то $2\leqslant n_1$ и $3\leqslant n_2$ и продолжим
$\leqslant\frac{9}{4}a^k<4^1\cdot a^k<(4\cdot a)^{2^k-1}$,
ибо $1<2^k-1$ и $k<2^k-1$ при $k\geqslant 2$.
То есть $\prod_{i=1}^kn_i<(4\cdot a)^{2^k-1}$.

 Профиль  
                  
 
 
Сообщение03.10.2008, 10:14 
Аватара пользователя


28/09/08
11
ddn писал(а):
$\leqslant\frac{9}{4}a^k$,

Я так понимаю это ваша лучшая оценка, но если{n_i=i+1} и a=1,b=k+1. Тогда что (k+1)!\leqslant\frac{9}{4}

Добавлено спустя 9 минут 32 секунды:

ddn писал(а):

Тогда $\prod^k_{i=1}n_i \leqslant n_1\prod_{i=2}^{k}\left(b\prod_{j=1}^{i-1}\left(1-\frac{1}{n_j}\right)\right)\leqslant n_1\cdot b\left(1-\frac{1}{n_1}\right)a^{k-2}<$.

Совершенно не понятно почему a может ограничевать сверху {n_i}.( видно что именно так вы и поступали).Левое неравенство в условии ,совершенно очевидно, не верно для каждого i . Хотя конечно ошибки на этом не заканчиваются.

 Профиль  
                  
 
 
Сообщение03.10.2008, 15:10 


06/07/07
215
СОКАКУ писал(а):
ddn писал(а):
$\leqslant\frac{9}{4}a^k$
Я так понимаю это ваша лучшая оценка, но если $n_i=i+1$ и $a=1$, $b=k+1$. Тогда что $(k+1)!\leqslant\frac{9}{4}$
$\prod_{i=1}^k\left(1-\frac{1}{n_i}\right)=\prod_{i=1}^k\frac{i}{i+1}=\frac{1}{k+1}$,
$\prod_{i=1}^{k-1}\left(1-\frac{1}{n_i}\right)=\prod_{i=1}^k\frac{i}{i+1}=\frac{1}{k}$,
$\frac{a}{b}=\frac{1}{k+1}$
Хм...

СОКАКУ писал(а):
ddn писал(а):
Тогда $\prod^k_{i=1}n_i \leqslant n_1\prod_{i=2}^{k}\left(b\prod_{j=1}^{i-1}\left(1-\frac{1}{n_j}\right)\right)\leqslant n_1\cdot b\left(1-\frac{1}{n_1}\right)a^{k-2}<$.

Совершенно не понятно почему a может ограничивать сверху {n_i}.( видно что именно так вы и поступали).Левое неравенство в условии ,совершенно очевидно, не верно для каждого i. Хотя конечно ошибки на этом не заканчиваются.
Этот вывод опирался на:
$n_i\leqslant b\prod_{j=1}^{i-1}\left(1-\frac{1}{n_j}\right)\leqslant a$ при $i\geqslant 3$.
Теперь вижу, что в условии $\prod_{i=1}^k\left(1-\frac{1}{n_i}\right)\leqslant\frac{a}{b}$ неравенство стоит для фиксированного $k$, а не любого $k\geqslant 2$ (иначе условие противоречиво).
То есть $b\prod_{j=1}^{i-1}\left(1-\frac{1}{n_j}\right)\leqslant a$ применимо лишь для $i=k+1$.


Тогда идем первоначальным путем
$\prod^k_{i=1}n_i \leqslant n_1\prod_{i=2}^{k}\left(b\prod_{j=1}^{i-1}\left(1-\frac{1}{n_j}\right)\right)=n_1b^{k-1}\prod_{j=1}^{k-1}\left(1-\frac{1}{n_j}\right)^{k-j-1}\leqslant$
$\leqslant\left(1-\frac{1}{n_1}\right)b^k\left(1-\frac{1}{n_k}\right)\prod_{j=1}^{k}\left(1-\frac{1}{n_j}\right)^{k-j-1}\leqslant\left(1-\frac{1}{n_1}\right)a^k\left(1-\frac{1}{n_k}\right)\prod_{j=1}^{k}\left(1-\frac{1}{n_j}\right)^{-j-1}<$

где $\left(1-\frac{1}{n_j}\right)^{-j-1}=e^{-(j+1)\ln\left(1-\frac{1}{n_j}\right)}<e^{\frac{j+1}{n_j-1}}$, так как
$\frac{1}{n}=\frac{1}{n}\cdot\frac{1}{1}<-\ln(1-\frac{1}{n})=\int\limits_{1-\frac{1}{n}}^{1}\frac{dx}{x}<\frac{1}{n}\cdot\frac{1}{1-\frac{1}{n}}=\frac{1}{n-1}$ при $n>1$

$<a^k\left(1-\frac{1}{n_1}\right)\left(1-\frac{1}{n_k}\right)e^{\sum\limits_{j=1}^{k}\frac{j+1}{n_j-1}}\leqslant a^ke^{k+\sum\limits_{j=1}^{k}\frac{1}{j}}<ka^ke^{k+1}=$
$=(e\cdot\sqrt[k]{ek}\cdot a)^k\leqslant(e\cdot e\cdot a)^k<(4^{\frac{1}{\ln(2)}}\cdot a)^k<(4\cdot a)^{2^k-1}$

учли еще, что $n_j\geqslant j+1$, $\sum\limits_{j=1}^{k}\frac{1}{j}<1+\ln(k)$, $\sqrt[k]{ek}\leqslant\sqrt[1]{e}=e$ и $\frac{k}{\ln(2)}<2^k-1$


Еще моя ошибка в том, что условие
$\prod_{i=1}^{k-1}\left(1-\frac{1}{n_i}\right)-\prod_{i=1}^k\left(1-\frac{1}{n_i}\right)\geqslant\frac{1}{b}$,
в действительности лишь достаточно (но не необходимо) для существования $a$, требуемых условием.
Оно лишь гарантирует их наличие, но требуемые $a$ могут существать (и существуют, как показал Ваш пример) и при $\prod_{i=1}^{k-1}\left(1-\frac{1}{n_i}\right)-\prod_{i=1}^k\left(1-\frac{1}{n_i}\right)<\frac{1}{b}$
(в вашем примере $...-...=\frac{1}{k(k+1)}<\frac{1}{k+1}=\frac{1}{b}$).

Неравенство $\prod^k_{i=1} n_i \leq (4 \cdot a)^{2^k - 1}$, таким образом, доказано для более узкой области значений $(a,b)$, чем требуется.
Если данное неравенство и верно при $\prod_{i=1}^{k-1}\left(1-\frac{1}{n_i}\right)-\prod_{i=1}^k\left(1-\frac{1}{n_i}\right)<\frac{1}{b}$,
то его доказательство существенно усложняется дискретностью значений $a$.
Думаю, без глубоких теорем теории чисел здесь ничего не доказать.

 Профиль  
                  
 
 
Сообщение05.10.2008, 08:16 
Заслуженный участник


09/02/06
4382
Москва
Мне кажется надо доказать $n_i\le (4a)^{2^{i-1}}$ по индукции. Вроде можно получить более точные оценки.

 Профиль  
                  
Показать сообщения за:  Поле сортировки  
Начать новую тему Ответить на тему  [ Сообщений: 9 ] 

Модераторы: Модераторы Математики, Супермодераторы



Кто сейчас на конференции

Сейчас этот форум просматривают: нет зарегистрированных пользователей


Вы не можете начинать темы
Вы не можете отвечать на сообщения
Вы не можете редактировать свои сообщения
Вы не можете удалять свои сообщения
Вы не можете добавлять вложения

Найти:
Powered by phpBB © 2000, 2002, 2005, 2007 phpBB Group